LSAT and Law School Admissions Forum

Get expert LSAT preparation and law school admissions advice from PowerScore Test Preparation.

User avatar
 Dave Killoran
PowerScore Staff
  • PowerScore Staff
  • Posts: 5853
  • Joined: Mar 25, 2011
|
#88993
Complete Question Explanation
(The complete setup for this game can be found here: lsat/viewtopic.php?f=237&p=88987)

The correct answer choice is (A)

If H does not sit next to G, and from the second rule H does not sit next to F, then H can only be seated midway between F and G:

G4-Q20-d1.png

Whichever side H is on, K will be on the opposite side (from the second rule), along with the OI block (the OI block cannot be on the H side as there is insufficient room). With K, O, and I on one side, the H side must also include M and P, with H between M and P.

Because P will never be next M, P cannot be listed among the pair of variables that are both next to M. Thus, you can immediately eliminate answer choices (B), (D), and (E).

With only (A) and (C) remaining, examine just one answer. If it is possible, choose this answer. If it is not, choose the other answer. Let us examine (A). Reading counterclockwise from F, the diagram would be F M H P G and (OI, K) in some order. This is possible, so answer choice (A) is correct. Answer choice (C) cannot occur because K and M are on opposite sides of the table.
You do not have the required permissions to view the files attached to this post.

Get the most out of your LSAT Prep Plus subscription.

Analyze and track your performance with our Testing and Analytics Package.